Integral ∫1−11x1+x1−x−−−√ln(2x2+2x+12x2−2x+1)dx∫−111x1+x1−xln⁡(2x2+2x+12x2−2x+1)dx\int_{- 1}^1\frac1x\sqrt{\frac{1+x}{1-x}}\ln\left(\frac{2\,x^2+2\,x+1}{2\,x^ 2-2\,x+1}\right) \mathrm dx

Ich brauche Hilfe bei diesem Integral:

ICH = 1 1 1 X 1 + X 1 X ln ( 2 X 2 + 2 X + 1 2 X 2 2 X + 1 )   D X .

Der Integrand-Graph sieht so aus:

Der Integrandengraph

Der ungefähre Zahlenwert des Integrals:

ICH 8.372211626601275661625747121...

Weder Mathematica noch Maple konnten eine geschlossene Form für dieses Integral finden, und Suchen des ungefähren numerischen Werts in WolframAlpha und ISC+ ergaben auch keine plausiblen Kandidaten für geschlossene Formen. Aber ich hoffe trotzdem, dass es vielleicht eine geschlossene Form dafür gibt.

Mich interessieren auch Fälle, in denen unter dem Logarithmus nur Zähler oder nur Nenner steht.

Haben Sie Grund zu der Annahme, dass es eine geschlossene Form für dieses schrecklich aussehende Ding gibt?
Mittlerweile konnte ich das Integral in folgende Form manipulieren:
8 0 D u ( u 2 1 ) ( u 4 6 u 2 + 1 ) u 8 + 4 u 6 + 70 u 4 + 4 u 2 + 1 Protokoll u
woraus ich schließen kann, dass es tatsächlich eine geschlossene Form gibt (weil die Wurzeln des Nenners in geschlossener Form ausdrückbar sind, ein wenig chaotisch, aber nicht schlecht). Aber da es acht Wurzeln gibt, wird ein auf Resten basierender Ansatz ohne etwas Computeralgebra sehr, sehr chaotisch und fast hoffnungslos sein.
@MhenniBenghorbal: Wie verbunden?
@ user88377 Warum gibt es dafür ein Kopfgeld? Es scheint, dass Ron Gordon bereits eine vollständige Lösung bereitgestellt hat ... Suchen Sie nach anderen Lösungen?
Aus Neugier, gibt es eine Anwendung für dieses Integral?
@Integrals Die Leute setzen Kopfgelder auf die Frage, nur um sie ihm zu geben, außer dem OP
Ich bin zufällig auf Ron Gordons nachträgliche Einfälle auf Reddit gestoßen und hätte sonst nichts davon gewusst: old.reddit.com/r/math/comments/1qpus4/master_of_integration/… . Ich glaube nicht, dass es hier verlinkt wurde? Es wäre so schade, sich dessen nicht bewusst zu sein!
Woher hast du dieses Problem? .. Ich bin sicher, wer dieses Problem gemacht hat, kennt irgendwie die Lösung oder einen Hintergrund, weil es nicht wie eine zufällige Formel aussieht.

Antworten (10)

Ich werde das Integral durch eine Substitution transformieren, es in zwei Teile zerlegen und neu kombinieren, eine partielle Integration durchführen und eine weitere Substitution durchführen, um ein Integral zu erhalten, von dem ich weiß, dass es eine geschlossene Form gibt. Von dort verwende ich eine mir bekannte Methode, um das Integral anzugreifen, aber auf ungewöhnliche Weise wegen des Polynoms 8. Grades im Nenner des Integranden.

Erste Sub T = ( 1 X ) / ( 1 + X ) , D T = 2 / ( 1 + X ) 2 D X zu bekommen

2 0 D T T 1 / 2 1 T 2 Protokoll ( 5 2 T + T 2 1 2 T + 5 T 2 )

Verwenden Sie nun die Symmetrie aus der Karte T 1 / T . Brechen Sie das Integral wie folgt in zwei Teile auf:

2 0 1 D T T 1 / 2 1 T 2 Protokoll ( 5 2 T + T 2 1 2 T + 5 T 2 ) + 2 1 D T T 1 / 2 1 T 2 Protokoll ( 5 2 T + T 2 1 2 T + 5 T 2 ) = 2 0 1 D T T 1 / 2 1 T 2 Protokoll ( 5 2 T + T 2 1 2 T + 5 T 2 ) + 2 0 1 D T T 1 / 2 1 T 2 Protokoll ( 5 2 T + T 2 1 2 T + 5 T 2 ) = 2 0 1 D T T 1 / 2 1 T Protokoll ( 5 2 T + T 2 1 2 T + 5 T 2 )

Sub T = u 2 zu bekommen

4 0 1 D u 1 u 2 Protokoll ( 5 2 u 2 + u 4 1 2 u 2 + 5 u 4 )

Teileweise integrieren:

[ 2 Protokoll ( 1 + u 1 u ) Protokoll ( 5 2 u 2 + u 4 1 2 u 2 + 5 u 4 ) ] 0 1 32 0 1 D u ( u 5 6 u 3 + u ) ( u 4 2 u 2 + 5 ) ( 5 u 4 2 u 2 + 1 ) Protokoll ( 1 + u 1 u )

Ein letztes Sub: u = ( v 1 ) / ( v + 1 ) D u = 2 / ( v + 1 ) 2 D v , und endlich bekommen

8 0 D v ( v 2 1 ) ( v 4 6 v 2 + 1 ) v 8 + 4 v 6 + 70 v 4 + 4 v 2 + 1 Protokoll v

Mit dieser Form können wir schließlich schlussfolgern, dass eine geschlossene Form existiert, und den Residuensatz anwenden, um sie zu erhalten. Betrachten Sie dazu das folgende Konturintegral:

C D z 8 ( z 2 1 ) ( z 4 6 z 2 + 1 ) z 8 + 4 z 6 + 70 z 4 + 4 z 2 + 1 Protokoll 2 z

Wo C ist eine Schlüssellochkontur um die positive reelle Achse. Dieses Konturintegral ist gleich (ich lasse die Schritte weg, in denen ich zeige, dass das Integral um die Kreisbögen verschwindet)

ich 4 π 0 D v 8 ( v 2 1 ) ( v 4 6 v 2 + 1 ) v 8 + 4 v 6 + 70 v 4 + 4 v 2 + 1 Protokoll v + 4 π 2 0 D v 8 ( v 2 1 ) ( v 4 6 v 2 + 1 ) v 8 + 4 v 6 + 70 v 4 + 4 v 2 + 1

Es sei darauf hingewiesen, dass das zweite Integral verschwindet; Dies kann leicht durch Ausnutzen der Symmetrie um gesehen werden v 1 / v .

Andererseits ist das Konturintegral ich 2 π mal der Summe der Residuen um die Pole des Integranden. Im Allgemeinen erfordert dies, dass wir die Nullstellen des Polynoms achter Ordnung finden, was analytisch möglicherweise nicht möglich ist. Hier hingegen haben wir viele Symmetrien auszunutzen, zB if A ist dann eine Wurzel 1 / A ist eine Wurzel, A ist eine Wurzel, und A ¯ ist eine Wurzel. Das können wir zum Beispiel ableiten

z 8 + 4 z 6 + 70 z 4 + 4 z 2 + 1 = ( z 4 + 4 z 3 + 10 z 2 + 4 z + 1 ) ( z 4 4 z 3 + 10 z 2 4 z + 1 )

die ausnutzt A A Symmetrie. Schreibe jetzt

z 4 + 4 z 3 + 10 z 2 + 4 z + 1 = ( z A ) ( z A ¯ ) ( z 1 A ) ( z 1 A ¯ )

Schreiben A = R e ich θ und erhalte folgende Gleichungen:

( R + 1 R ) cos θ = 2
( R 2 + 1 R 2 ) + 4 cos 2 θ = 10

Aus diesen Gleichungen kann man ableiten, dass eine Lösung ist R = ϕ + ϕ Und cos θ = 1 / ϕ , Wo ϕ = ( 1 + 5 ) / 2 ist der Goldene Schnitt. So nehmen die Pole die Form an

z k = ± ( ϕ ± ϕ ) e ± ich arctan ϕ

Nun müssen wir die Reste des Integranden an diesen 8 Polen finden. Wir können diese Aufgabe aufteilen, indem wir Folgendes berechnen:

k = 1 8 Auflösung z = z k [ 8 ( z 2 1 ) ( z 4 6 z 2 + 1 ) Protokoll 2 z z 8 + 4 z 6 + 70 z 4 + 4 z 2 + 1 ] = k = 1 8 Auflösung z = z k [ 8 ( z 2 1 ) ( z 4 6 z 2 + 1 ) z 8 + 4 z 6 + 70 z 4 + 4 z 2 + 1 ] Protokoll 2 z k

Hier wurde es sehr chaotisch, aber das Ergebnis ist ziemlich einfach:

Auflösung z = z k [ 8 ( z 2 1 ) ( z 4 6 z 2 + 1 ) z 8 + 4 z 6 + 70 z 4 + 4 z 2 + 1 ] = Zeichen [ cos ( Arg z k ) ]

BEARBEITEN

Eigentlich ist dies eine sehr einfache Berechnung. Inspiriert von @sos440 kann man die rationale Funktion von ausdrücken z in ganz einfacher Form:

8 ( z 2 1 ) ( z 4 6 z 2 + 1 ) z 8 + 4 z 6 + 70 z 4 + 4 z 2 + 1 = [ P ' ( z ) P ( z ) + P ' ( z ) P ( z ) ]

Wo

P ( z ) = z 4 + 4 z 3 + 10 z 2 + 4 z + 1

Die Reste dieser Funktion an den Polen sind dann leicht zu erkennen ± 1 je nachdem, ob der Pol eine Null von ist P ( z ) oder P ( z ) .

ENDE BEARBEITEN

Das heißt, wenn der Pol einen positiven Realteil hat, ist der Rest des Bruchs + 1 ; wenn es einen negativen Realteil hat, ist das Residuum 1 .

Betrachten Sie nun das Protokollstück. Wenn wir das Quadrat erweitern, erhalten wir 3 Terme:

Protokoll 2 | z k | ( Arg z k ) 2 + ich 2 Protokoll | z k | Arg z k

Summiert man die Reste, so findet man das wegen der ± 1 Beiträge oben, dass die Summe der ersten und dritten Terme Null ist. Damit bleibt die zweite Amtszeit. Dafür ist es entscheidend, dass wir die richtigen Argumente finden, wie Arg z k [ 0 , 2 π ) . Somit haben wir

ICH = 0 D v 8 ( v 2 1 ) ( v 4 6 v 2 + 1 ) v 8 + 4 v 6 + 70 v 4 + 4 v 2 + 1 Protokoll v = 1 2 k = 1 8 Zeichen [ cos ( Arg z k ) ] ( Arg z k ) 2 = 1 2 [ 2 ( arctan ϕ ) 2 + 2 ( 2 π arctan ϕ ) 2 2 ( π arctan ϕ ) 2 2 ( π + arctan ϕ ) 2 ] = 2 π 2 4 π arctan ϕ = 4 π arccot ϕ

+1. arccot ( ϕ ) -Definitiv eine der seltsamsten Closed-Form-Lösungen, die jemals erhalten wurden!
@ShikariShambu: Ich frage mich, ob wir eine Liste der seltsamsten Lösungen in geschlossener Form für Integrale, Summen, Produkte, Diff-Gleichungen und dergleichen erstellen können. Was macht ein geschlossenes Formular "seltsam"? Was macht diesen seltsam? Die Gegenüberstellung des Arkuskotangens und ϕ ? Das könnte zu einer seltsamen Diskussion führen, die aber trotzdem Spaß macht.
ich weiß, dass ϕ hat einige nette Eigenschaften, aber am Ende des Tages ist es nur eine weitere algebraische Zahl:
ϕ = 1 + 5 2 ,
und alles, was wir getan haben, ist, den Bogenkotangens der Quadratwurzel davon zu nehmen und mit zu multiplizieren 4 π . - Ich frage mich nur, warum dieses Ergebnis als "seltsam" angesehen wird. Es erscheint höchstwahrscheinlich, weil es zufällig eine Wurzel eines Polynoms ist, das hat ( 1 + 5 ) / 2 als eine seiner Wurzeln.
@pbs: Was für eine Person seltsam ist, ist es für eine andere nicht. Nur eine Frage des Geschmacks. In diesem Fall eine Frage der Erwartungen. War die Anwesenheit von ϕ oder seine Quadratwurzel, die Ihnen offensichtlich war, als Sie sich diesem Problem zum ersten Mal genähert haben? Arctan oder Arccot, je nachdem, wie Sie das Ergebnis ausgedrückt haben? Es war für mich alles andere als offensichtlich, oder dass wir einen so schönen, kompakten Ausdruck für ein Ergebnis haben würden.
@RonGordon Stimmt, und nein, es wurde sicherlich nicht erwartet! Andererseits erwarte ich nicht, dass eine bestimmte algebraische Zahl in einem Ergebnis erscheint, bis sie erscheint, also denke ich, dass die Überraschung hier ist ϕ hat eine besondere Bedeutung. Ich mag Ihre Vorstellung von Erwartungen.
@RonGordon Wenn ich solche Dinge sehe, halte ich es weniger für seltsam als für eine angenehme Überraschung. Phi ist aus vielen Gründen als Schönheit bekannt, und es zu sehen, wie es auftaucht, um unseren Freund dfeuer zu zitieren, „dieses schrecklich aussehende Ding“, ist in der Tat eine ziemlich angenehme Überraschung! Keine Stanniolhüte, keine Kubikwurzel-aus-31-Verschwörungen, nur einen alten Freund im Lebensmittelgeschäft treffen =)
@corsiKa: ϕ allein, sicher. Aber jetzt stell dir vor, du triffst deinen alten Freund, aber gekleidet in Frauenkleidern mit einem Militärhelm aus der Kaiserzeit, Spikes und allem. Das ist das Gefühl, das man bekommt, wenn man sieht, nicht normal alt ϕ , aber ARCCOT SQRT ϕ .
Ich hoffe, Sie haben nichts dagegen, @RonGordon, Sir, ich möchte Ihnen +50 Wiederholungen geben; Die beispielhafte Antwort ist sowohl aufschlussreich als auch inspirierend. Darin steht, dass ich Ihnen das Kopfgeld in 23 Stunden geben kann; Ich werde das Kopfgeld geben, wenn es möglich ist.
Sir, können Sie erklären, was Sie meinen, wenn Sie „die Symmetrie aus der Karte verwenden“. T 1 / T " ?
@onurcanbektas In 1 D T T 1 / 2 1 T 2 Protokoll ( 5 2 T + T 2 1 2 T + 5 T 2 ) , Ändern T Zu 1 / T im Argument zu Protokoll bewirkt das Invertieren von Zähler und Nenner, also insgesamt Protokoll term nimmt einfach ein Minuszeichen auf. Das Integral ändert sich also kaum unter der Änderung von Variablen T = 1 / u , D T = D u / u 2 : du erhältst 0 1 D u u 1 / 2 1 u 2 Protokoll ( 5 2 u + u 2 1 2 u + 5 u 2 ) . Sie können dann wiederherstellen T als Integrationsvariable und rekombiniere die Integrale.
@ConnorHarris Oh, ich verstehe. Danke für die Erklärung.
Ich bin zufällig auf Ihre nachträglichen Gedanken auf Reddit gestoßen und hätte sonst nichts davon gewusst: old.reddit.com/r/math/comments/1qpus4/master_of_integration/… . Hast du es hier verlinkt? Es wäre so schade, sich dessen nicht bewusst zu sein!
Dies ist sehr alt, aber BITTE empfehlen Sie mir einige Lernmaterialien, die mir helfen, ein solcher Meister der Integration zu werden.

ICH = 4 π arccot ϕ

@OliverBel IMHO, wenn die Frage nicht explizit nach einem Beweis fragt und es keine offensichtliche vermutete geschlossene Form gibt, denke ich, dass es in Ordnung ist, das Ergebnis zuerst zu posten und später einen Beweis hinzuzufügen, wenn es die Zeit erlaubt. Das Ergebnis selbst kann für die fragende Person nützlich sein. Es kann heruntergestimmt werden, wenn es nicht nützlich ist, aber es gilt immer noch als Antwort. Die Richtlinie für Kommentare lautet "Vermeiden Sie die Beantwortung von Fragen in Kommentaren", und ich sehe keinen Grund, das Ergebnis zurückzuhalten, bis der vollständige Beweis zur Veröffentlichung bereit ist.
@VladimirReshetnikov: Ich verweise auf Hamming: "Der Zweck des Rechnens ist Einsicht, nicht Zahlen." Wenn das Ergebnis selbst nicht besonders aufschlussreich ist, stimme ich nicht zu, dass es eine Antwort ist. Es gibt einen Benutzer, der weiterhin darauf besteht, die Maple-Ausgabe als Antwort zu verwenden, und ich denke, das verschlechtert die Site. Ich würde es vorziehen, wenn Cleo einen Einblick geben würde, woher diese Antwort kam.
@RonGordon Natürlich würde ich auch lieber den Beweis sehen. Ich hoffe, dass Cleo es irgendwann postet oder jemand anderes es tut, inspiriert von der Tatsache, dass die geschlossene Form existiert (und vielleicht auch von ihrer Form geleitet). Mein Punkt ist, dass es (zumindest bei mir) Stunden dauern kann, einen Korrekturabzug zu schreiben und zu setzen, und ich kann mir vorstellen, dass nicht jeder die erforderliche Zeit zeitnah aufbringen kann. Am liebsten würde ich in solchen Fällen zumindest zeitnah das Ergebnis gepostet sehen. (Und aus meiner Erfahrung als Doktorand in Theoretischer Physik kann ich sagen, dass manchmal das Ergebnis das einzige ist, was eine Person interessiert :)
@VladimirReshetnikov: Gut, aber wir sollten Cleo bitten, zumindest etwas darüber zu sagen, wie er das bekommen hat und dass ein Beweis in Arbeit ist. Vielleicht hat er Mathematica manipuliert, um die Antwort auf eine Weise zu erzeugen, die das OP nicht konnte. (Das ist immer noch nicht gut genug für eine Antwort, aber es ist zumindest gut genug für einen Kommentar.) Aber nur ein Ergebnis einzufügen, das mit den Zahlen übereinstimmt, ist meiner Meinung nach nicht gut genug für diese Site.
Ich stimme Ihnen beiden zu, die Antwort ist an sich schön - ich würde fast sagen, unglaublich schön angesichts des Verhältnisses von Einfachheit des Ergebnisses zu Komplexität des Problems, aber wir lernen nichts aus dieser Antwort, Manipulation eines CAS ist eine nützliche Fähigkeit, wenn man diese Probleme angeht, und ich würde gerne etwas sehen , das mir sagt, wie ich solche Dinge angehen soll.
Angenommen, diese Antwort ist richtig (was sie sicherlich mit vielen Dezimalstellen ist), erscheint es sehr seltsam. π Manchmal erscheint ein Arkuskotangens seltsam genug. Werfen der Quadratwurzel von ϕ hinein scheint noch seltsamer.
@dfeuer: Ich kann dir sagen, wo der Platz ist ϕ kommt von: es ist die Phase der Wurzeln der Oktik (Wort?) im Integranden, den ich in den Kommentaren zur Frage gepostet habe. Den Rest arbeite ich noch aus.
Hey @RonGordon, was meinst du mit "Phase der Wurzeln"? Ich sehe, die Wurzeln sind von der Form
u = ( ± 1 ± ich ) ± 1 ± 2 ich
(nahe genug?)
@Bennett: Das stimmt, soweit ich das von meinem Handy aus beurteilen kann.
Was ist also diese „Phase“? Ist mir nicht geläufig.
@BennettGardiner Ich nehme an, die Phase ist Arg z .
@BennettGardiner: Vladimir hat recht. Weitere folgen.
@VladimirReshetnikov: ein anderer Gedanke. Ich denke, Sie sollten sich darum kümmern, wie wir hier die Antworten bekommen. Ich hoffe, dass Sie von dem von mir beschriebenen Prozess profitieren konnten, um ans Ziel zu gelangen. Wie werden wir ohne die Straße jemals verstehen, wie wir ans Ziel kommen? Und wie werden wir jemals eine bessere Straße finden? Daher betrachte ich Cleos "Antworten" nicht als solche. Cleo hat vier solcher Antworten ohne Hinweis auf einen Denkprozess gepostet. Soweit ich weiß, hat er möglicherweise nur einen Weg gefunden, Maple zu manipulieren, um eine plausible Antwort auszugeben. Aber ich habe keine Ahnung.
Diese Art der Antwort ist völlig respektlos. Diese Situation sieht für mich so aus: Cleo hat ein interessantes Problem gefunden und es gelöst. Er ist faul, die Lösung zu schreiben, möchte aber zeigen, wie schlau er ist, also hat er beschlossen, nur das Endergebnis zu posten. Der Hinweis auf die Definition des Goldenen Schnitts brachte mich zum Lachen. Wenn OP eine solche Frage stellt, ist er definitiv mit dieser Konstante vertraut. Beachten Sie, dass dies kein einzelnes Beispiel ist. ALLE Antworten von Cleo sind in diesem Stil, und selbst nach höflicher Betonung, dass diese Antworten nicht das sind, was OPs wollten, veröffentlicht er weiterhin nur Endergebnisse!
@Norbert: Dem stimme ich vollkommen zu. Ich habe keine Ahnung, was die Leute dazu bringt, diese Antwort positiv zu bewerten (vielleicht eine sklavische Hingabe, die erste Antwort positiv zu bewerten, von der ich weiß, dass sie bei bestimmten Benutzern weit verbreitet ist). Diese Antworten sind keine solchen und sollten nicht nur energisch abgelehnt, sondern auch als Nicht-Antworten gekennzeichnet werden.
@Norbert Ich würde es nicht zu persönlich nehmen. Die Antwort ist definitiv nicht anstößig. Sicherlich ist es von geringer Qualität und kann von denen abgelehnt werden, die es für nicht nützlich halten.
Ich werde meine Meinung nicht ändern, da ich den Stil dieses Benutzers kenne.
Können Sie uns einen Hinweis geben, wie Sie zu Ihrer Lösung gekommen sind? Welches Programm hast du verwendet?
@Norbert Meiner Meinung nach ist dies keine kluge Antwort, dies ist eine arrogante Antwort und ich verstehe nicht, warum sie so viele positive Stimmen hat. Eine Erklärung zu dieser Lösung des Integrals ist erforderlich.
«Respektlose Antwort» und «arrogante Antwort»? Wirklich?! Ich finde die Antwort ziemlich nutzlos, da ich nichts daraus gelernt habe, aber respektlos und arrogant sind Urteile, die dieser Antwort völlig unangemessen erscheinen! Ich wünschte wirklich , die Kommentatoren würden die dramatische Aufladung ihrer Kommentare auf den Wert eines Integrals beschränken!
@Norbert: Aus Cleos Benutzerprofil: "Ich habe eine Krankheit, die es mir sehr schwer macht, mich an Gesprächen zu beteiligen oder lange Antworten zu posten, sorry dafür." Möchten Sie Ihr Urteil überdenken?
@EdwardKMETT, du bist Neuling hier. Glauben Sie mir, Sie kennen die Situation nicht im Detail. Was den Gesundheitszustand angeht... das ist eine Lüge, denn die erste Erklärung, die Cleo vor langer Zeit gegeben hat, war... Ratet mal, was? Das ist ihre Religion, nackte Antworten zu geben, und wenn jemand an der Richtigkeit zweifelt, dann sollte man diese Antwort zu seinem Axiomensystem hinzufügen. Das ist lächerlich!
@Norbert :'( Nun, ich hoffe, wir haben den Urlaub dieses Benutzers nicht verursacht. Er/sie war extrem wertvoll und ich liebe viel von ihrer Arbeit.
@SimplyBeautifulArt: Tut mir leid, dass deine Blase geplatzt ist. Bei wahrer Mathematik geht es nicht darum, egoistisch zu sein und seine Methoden für sich zu behalten. Außerdem lügt er/sie absichtlich, um die Menschen dazu zu bringen, ihn/sie zu bemitleiden und mathematisch leere Antworten zuzulassen, was dem Geist der Mathematik bei der Suche nach Wahrheit (oder zumindest Einsicht) widerspricht. Woher wissen Sie, dass alle seine/ihre Antworten wahr sind, da er/sie über Motive lügt?? Siehe meta.math.stackexchange.com/q/11723 und meta.math.stackexchange.com/questions/11759/… .
@user21820 Natürlich glaube ich nicht, dass ihre Antworten die besten für die Seite sind, aber ich glaube auch, dass die talentierten Di der Seite in gewisser Weise zugute kommen, ob egoistisch oder nicht. Ich mag auch nicht den Gedanken, dass Benutzer mit anständig hohen Wiederholungszahlen uns verlassen. Es macht mich traurig zu sehen.
@SimplyBeautifulArt: Beachten Sie Folgendes: (1) Einzeilige Antworten sollten in fast allen Fällen nur ein Kommentar sein. (2) Hohes Ansehen bedeutet die Fähigkeit, im SE-Umfeld ein hohes Ansehen zu erlangen, und impliziert keineswegs die Qualität ihrer Beiträge. Es gibt viele Benutzer mit hohem Ansehen, die dadurch gewonnen wurden, dass sie jeden Tag so viele grundlegende Fragen zur Analysis beantworten, wie sie nur können, die so schlecht mit Logik umgehen können, dass sie auch Unsinn von sich geben. (3) Dieser Benutzer hat keine Reputation, wenn alle seine/ihre mathematisch leeren Beiträge entfernt werden!
68 Stimmen dafür, nur eine Antwort zu posten....
@imranfat, denkst du, jemand würde es wagen, sich dieses Integral anzusehen, ohne zu wissen, dass es eine schöne geschlossene Form dafür gibt? Diese Antwort ist der halbe Weg zur Lösung des Integrals. Diese Frage gewann durch das interessante geschlossene Formular, das sein Benutzer zur Verfügung stellte, an Popularität.
@ZaidAlyafeai. Ok, warten wir auf "die andere Hälfte", dann ruhe ich mich aus :)
@ZaidAlyafeai Ich stimme von ganzem Herzen zu. Es gibt eine Menge Integrale auf dieser Seite ohne Antworten, weil keine geschlossene Form bekannt ist, und so geben die Leute auf, wenn sie keine finden können. Eine Antwort zu haben ist zwar nicht alles, aber es ist ziemlich wichtig zu wissen, dass ein Integral lösbar ist und es daher wert ist, Zeit damit zu verbringen, es zu lösen. Darüber hinaus kann die Kenntnis der Lösung oft ein Schritt in Richtung ihrer Lösung sein. Sind Proofs in 99% der Fälle besser? Ja. Viel Glück beim Erhalten dieser Beweise für eine Reihe unmöglich aussehender Integrale, ohne zu wissen, dass die Integrale zuerst lösbar sind: /
@BrevanEllefsen nehmen Sie dies zum Beispiel math.stackexchange.com/questions/2242141/… .
Gee Leute, keine Notwendigkeit für all dieses Händeringen; Stimmen Sie einfach nutzlose Antworten ab, ohne etwas zu sagen, und machen Sie weiter!
@user21820 Behaupten Sie, dass Cleo in seiner/ihrer Biografie über den Gesundheitszustand lügt? Wie sind Sie zu diesem Schluss gekommen?
@Blau: In der Tat. Ich bin zu dieser Schlussfolgerung gelangt, nachdem ich mich ausreichend bemüht hatte, die Behauptungen zahlreicher anderer Benutzer zu überprüfen, die Cleos früheres Verhalten beobachtet haben, wie z. B. das, was Norbert oben gesagt hat. Zuerst sagte sie, sie habe ihre Antworten von Namagiri bekommen, und dann, als sie weiter befragt wurde, sagte sie: „Es gibt viele Möglichkeiten, dieses Ergebnis zu beweisen. Die einfachste ist, in einem Axiomatiksystem zu arbeiten, das es als Axiom akzeptiert. Ich bevorzuge diesen Ansatz, wenn.“ Ich kenne das Ergebnis, daher wird hier der vollständige Beweis gegeben.“ um ihre beweislose Antwort zu rechtfertigen!! Später legte sie die Lüge in ihr Profil und löschte ihre Kommentare.
@ user21820 Oh, ich verstehe.
Warum vermuten so viele Benutzer zu Beginn dieses Kommentarthreads @RonGordon, Norbert ?? Ich stimme dem zu, was Sie hier posten ++++, aber ich bin verwirrt, was Ihre Argumentation betrifft, "Cleo" als "er" oder "sein" zu bezeichnen. Vor allem angesichts "ihres" Gravatars.
Ich hätte „sie“ verwenden sollen, weil ich Cleos Geschlecht nicht kannte. Ich entschuldige mich dafür.
@ user21820 Der traurige Teil der Situation ist, wenn sie herauskäme und zugab, dass sie diese mithilfe von Computerunterstützung löste (was wahrscheinlich erscheint), wäre sogar die Methode, mit der sie ein Computerprogramm verfeinerte, um diese Ergebnisse auszugeben, immens nützlich. Stattdessen entscheidet sie sich, aus uns unbekannten Gründen mysteriös zu bleiben.
@DonThousand: Genau. Diese Art von Verhalten ist wirklich widerlich.
Jahre später, und wir reden immer noch darüber. Wie auch immer man über die Tortur denkt, es ist einfach unpraktisch, sich 8 Jahre später darüber zu streiten. Cleo ist offenbar längst nicht mehr so ​​aktiv auf der Seite wie früher. Außerdem unterstützt ihre Antwort nicht die Vorstellung, dass „das Posten von Antworten ohne Beweis akzeptabel ist“. Niemand wird anfangen, häufig solche nackten Antworten zu posten und seine Taten mit „Cleo hat es geschafft, also kann ich es auch“ zu rechtfertigen, es ist ein rutschiger Abhang. Ich kann die gemeinsame Frustration der Benutzer von vor Jahren verstehen, aber 8 Jahre später entspricht der Punkt nicht mehr der gleichen moralischen Überzeugung.
Tschüss, Cleo! Es ist 2022 und du bist für mich zur Legende geworden. Möge deine Antwort für immer andauern.
Manchmal vermisse ich Cleos Antworten.

NEUE ANTWORT. Ich habe noch einen anderen Weg gefunden, dieses Problem zu lösen. Meine neue Lösung verwendet keine Konturintegration und basiert auf folgender Beobachtung: z | z | 1 ,

1 1 1 X 1 + X 1 X Protokoll ( 1 z X ) D z = π Sünde 1 z π Protokoll ( 1 2 + 1 2 1 z 2 ) .

Da ich sowohl die alte als auch die neue Antwort behalten möchte, habe ich meine neue Lösung auf einer anderen Seite gepostet. Sie können es hier überprüfen .


ALTE ANTWORT. Okay, hier ist eine andere Lösung. Es hängt auch mit meiner Verallgemeinerung zusammen .

Wir behaupten folgenden Satz:

Vorschlag. Wenn 0 < R < 1 Und R < S , Dann

(1) ICH ( R , S ) := 1 1 1 X 1 + X 1 X Protokoll ( 1 + 2 R S X + ( R 2 + S 2 1 ) X 2 1 2 R S X + ( R 2 + S 2 1 ) X 2 ) D X = 4 π arcsin R .

Unter der Annahme dieses Satzes brauchen wir nur das nichtlineare Gleichungssystem zu lösen

2 R S = 2 Und R 2 + S 2 1 = 2.

Die eindeutige Lösung, die die Bedingung des Satzes erfüllt, ist R = ϕ 1 Und S = ϕ . Also durch (1) wir haben

1 1 1 X 1 + X 1 X Protokoll ( 1 + 2 X + 2 X 2 1 2 X + 2 X 2 ) D X = ICH ( ϕ 1 , ϕ ) = 4 π arcsin ( ϕ 1 ) = 4 π arccot ϕ .

Es bleibt also der Satz zu beweisen.


Beweis des Satzes. Wir teilen den Beweis in mehrere Schritte auf.

Schritt 1. (Fallreduktion durch analytische Fortsetzung) Wir bemerken zunächst das Gegebene R Und S , wir haben immer

(2) Mindest 1 X 1 { 1 ± 2 R S X + ( R 2 + S 2 1 ) X 2 } > 0.

Tatsächlich ist es nicht schwer zu überprüfen, ob wir die folgende Gleichheit verwenden

1 ± 2 R S X + ( R 2 + S 2 1 ) X 2 = ( 1 ± R S X ) 2 ( 1 R 2 ) ( 1 S 2 ) X 2 .

Dann (2) zeigt, dass der Integrand von ICH ( R , S ) bleibt holomoprhic unter kleinen Störungen von S In C . So können wir uns erweitern S ICH ( R , S ) als holomorphe Funktion auf einer offenen Menge, die das Liniensegment enthält ( R , ) C . Dann genügt es, dies nach dem Prinzip der analytischen Fortsetzung zu beweisen (1) gilt für R < S < 1 .

Schritt 2. (Integraldarstellung von ICH ) setzen R = Sünde a Und S = Sünde β , Wo 0 < a < β < π 2 . Dann

ICH ( R , S ) = 1 1 1 + X X 1 X 2 Protokoll ( 1 + 2 R S X + ( R 2 + S 2 1 ) X 2 1 2 R S X + ( R 2 + S 2 1 ) X 2 ) D X = 0 1 2 X 1 X 2 Protokoll ( 1 + 2 R S X + ( R 2 + S 2 1 ) X 2 1 2 R S X + ( R 2 + S 2 1 ) X 2 ) D X (  Parität ) = 1 2 X 2 1 Protokoll ( X 2 + 2 R S X + ( R 2 + S 2 1 ) X 2 2 R S X + ( R 2 + S 2 1 ) ) D X ( X X 1 ) = 0 1 2 T Protokoll ( ( T + T 1 ) 2 + 4 R S ( T + T 1 ) + 4 ( R 2 + S 2 1 ) ( T + T 1 ) 2 4 R S ( T + T 1 ) + 4 ( R 2 + S 2 1 ) ) D T ,

wobei wir in der letzten Zeile die Substitution verwendet haben X = 1 2 ( T + T 1 ) . Wenn wir das quartische Polynom einführen

P ( T ) = T 4 + 4 R S T 3 + ( 4 R 2 + 4 S 2 2 ) T 2 + 4 R S T + 1 ,

dann bei der Unterkunft P ( 1 / T ) = T 4 P ( T ) , können wir vereinfachen

ICH ( R , S ) = 2 0 1 Protokoll P ( T ) Protokoll P ( T ) T D T = 0 Protokoll P ( T ) Protokoll P ( T ) T D T = 0 ( P ' ( T ) P ( T ) + P ' ( T ) P ( T ) ) Protokoll T D T = 1 2 ( P ' ( z ) P ( z ) + P ' ( z ) P ( z ) ) Protokoll z D z ,

wo wir den Astschnitt wählen Protokoll so, dass es die obere Halbebene vermeidet

H = { z C : z > 0 } .

Schritt 3. (Rückstandsberechnung) Seit

F ( z ) := ( P ' ( z ) P ( z ) + P ' ( z ) P ( z ) ) Protokoll z = Ö ( Protokoll z z 2 ) als  z ,

indem man die Integrationskontur durch einen Halbkreis mit genügend großem Radius ersetzt, folgt das

ICH ( R , S ) = 1 2 { 2 π ich z 0 H Auflösung z = z 0 F ( z ) } = π z 0 H Auflösung z = z 0 F ( z ) .

(Es stellt sich heraus, dass F ( z ) hat nur logarithmische Singularität am Ursprung. Der Wert wird also nicht berücksichtigt ICH ( R , S ) .) Aber durch eine einfache Rechnung, zusammen mit der Bedingung 0 < a < β < π 2 , stellen wir leicht fest, dass die Nullstellen von P ( z ) sind genau

e ± ich ( a + β ) Und e ± ich ( a β ) .

Nun lass Z + sei die Nullstellenmenge von P ( z ) In H Und Z sei die Nullstellenmenge von P ( z ) In H . Dann

Z + = { e ich ( β + a ) , e ich ( β a ) } Und Z = { e ich ( β + a ) , e ich ( β a ) } .

Das zeigt sich besonders

P ' ( z ) P ( z ) Protokoll z = z 0 Z + Protokoll z z z 0 + holomorphe Funktion ein  H

Und

P ' ( z ) P ( z ) Protokoll z = z 0 Z Protokoll z z z 0 + holomorphe Funktion ein  H .

Daraus folgt also

ICH ( R , S ) = π { z 0 Z + Protokoll z 0 z 0 Z Protokoll z 0 } = π { Protokoll e ich ( β + a ) + Protokoll e ich ( π β + a ) Protokoll e ich ( π β a ) Protokoll e ich ( β a ) } = π { ich ( β + a ) + ich ( π β + a ) ich ( π β a ) ich ( β a ) } = 4 π a = 4 π arcsin R .

Damit ist der Beweis abgeschlossen.

Sehr schön, und es hilft mir auch, einen Teil meines Beweises zu vereinfachen (ich denke, unsere Lösungen haben mehr gemeinsam als nicht). Eine Frage allerdings: Was ist mit dem Verzweigungspunkt des Protokolls in der Rückstandsberechnung? Ich weiß, dass es keine Rolle zu spielen scheint, da Sie am Ende die richtige Lösung erhalten, aber Sie möchten vielleicht etwas darüber sagen, wie Sie den Verzweigungspunkt am Ursprung vermeiden und einen Zweig des Protokolls definieren (was Sie meiner Meinung nach sowieso mit Ihrem tun Beschränkungen auf a Und β ) in der komplexen Ebene.
@RonGordon, Wie in der Lösung geschrieben, wird der Astschnitt des Baumstamms so gewählt, dass er die obere Halbebene vermeidet. Also wäre es sicher, wenn wir es als negative y-Achse wählen, aber ich denke, der Standard-Astschnitt ( , 0 ) würde auch funktionieren.
Oh je, da ist es. Meine Güte, tut mir leid. Auf jeden Fall hat mir das Lesen Ihrer Lösung geholfen, einen kleinen Teil meiner zu vereinfachen, also danke.

Unser Ziel ist es, einen elementaren Beweis der Satzformel (1) in der Antwort von @sos440 zu geben . Das stellen wir zunächst fest

Mindest 1 X 1 { 1 ± 2 R S X + ( R 2 + S 2 1 ) X 2 } > 0.
In der Tat, wenn X = ± 1 Dann
1 ± 2 R S X + ( R 2 + S 2 1 ) X 2 ( R S ) 2 > 0 ,
Wenn X = 0 Dann
1 ± 2 R S X + ( R 2 + S 2 1 ) X 2 = 1 > 0 ,
Wenn 1 < X < 1 , X 0 dann die Gleichungen
S ( 1 ± 2 R S X + ( R 2 + S 2 1 ) X 2 ) = 0 , R ( 1 ± 2 R S X + ( R 2 + S 2 1 ) X 2 ) = 0 ,
geben ± R = S X , ± S = R X , was unmöglich ist.

Im zweiten Schritt zeigen wir das ICH ( R , S ) ist unabhängig von S .

S ICH ( R , S ) = 1 1 1 + X 1 X 4 R ( 1 + ( R 2 S 2 1 ) X 2 ) ( 1 2 R S X + ( R 2 + S 2 1 ) X 2 ) ( 1 + 2 R S X + ( R 2 + S 2 1 ) X 2 D X .
Ersetzen X := X und indem wir sie addieren, erhalten wir
2 S ICH ( R , S ) = 1 1 2 1 X 2 4 R ( 1 + ( R 2 S 2 1 ) X 2 ) ( 1 2 R S X + ( R 2 + S 2 1 ) X 2 ) ( 1 + 2 R S X + ( R 2 + S 2 1 ) X 2 D X ,
das ist,
S ICH ( R , S ) = 1 1 1 1 X 2 4 R ( S 2 + R 2 1 ) X 2 + 4 R 1 + ( R 2 + S 2 1 ) 2 X 4 + ( 2 S 2 4 R 2 S 2 + 2 R 2 2 ) X 2 D X .
Ersetzen X := Sünde ( T ) wir haben
S ICH ( R , S ) = π / 2 π / 2 4 R ( S 2 + R 2 1 ) Sünde ( T ) 2 + 4 R 1 + ( R 2 + S 2 1 ) 2 Sünde ( T ) 4 + ( 2 S 2 4 R 2 S 2 + 2 R 2 2 ) Sünde ( T ) 2 D T
= π / 2 π / 2 8 R ( ( S 2 + R 2 1 ) cos ( 2 T ) + S 2 R 2 1 ) ( R 2 + S 2 1 ) 2 cos ( 2 T ) 2 2 ( R 2 S 2 1 ) ( R 2 + 1 S 2 ) cos ( 2 T ) + R 4 + ( 2 6 S 2 ) R 2 + ( S 2 + 1 ) 2 D T
= π π 4 R ( ( S 2 + R 2 1 ) cos ( j ) + S 2 R 2 1 ) ( R 2 + S 2 1 ) 2 cos ( j ) 2 2 ( R 2 S 2 1 ) ( R 2 + 1 S 2 ) cos ( j ) + R 4 + ( 2 6 S 2 ) R 2 + ( S 2 + 1 ) 2 D j .
Einführung in die neue Variable T := bräunen j 2 wir erhalten
S ICH ( R , S ) = 4 R ( S 2 R 2 ) T 2 4 R ( R S ) 2 ( R + S ) 2 T 4 + ( ( 2 4 S 2 ) R 2 + 2 S 2 ) T 2 + 1 D T = 4 R ( S 2 R 2 ) ( R S ) 2 ( R + S ) 2 T 2 + A T 4 + B T 2 + B 2 / 4 + D D T = 4 R ( S 2 + R 2 ) ( R S ) 2 ( R + S ) 2 ( 2 A ( B 2 + 4 D ) + ( B 2 + 4 D ) 3 / 2 ) π ( B 2 + 4 D ) 3 / 2 B 2 + 4 D + B ,
Wo
A = 1 S 2 R 2 ,
B = ( 2 4 S 2 ) R 2 + 2 S 2 ( R S ) 2 ( R + S ) 2 ,
B 2 + 4 D = 4 ( R S ) 2 ( R + S ) 2 .
Es gibt 2 A B 2 + 8 D A + ( B 2 + 4 D ) 3 / 2 = 0 .

Seit S ICH ( R , S ) = 0 wir haben

ICH ( R , S ) = ICH ( R , 1 ) = 1 1 1 X 1 + X 1 X Protokoll ( ( 1 + R X ) 2 ( 1 R X ) 2 ) D X .
Davon
R ICH ( R , 1 ) = 1 1 1 + X 1 X 4 1 R 2 X 2 D X .
Ähnlich wie oben erhalten wir
R ICH ( R , 1 ) = 1 1 4 1 X 2 ( 1 R 2 X 2 ) D X = 4 π 1 R 2 = 4 π ( arcsin R ) ' .
Es impliziert
ICH ( R , 1 ) = 4 π arcsin R + C .
Die Grenze nehmen lim R 0 + wir erhalten C = 0 , das ist, ICH ( R , S ) = 4 π arcsin R .

@ sos440 Danke. Es war keine triviale Aufgabe. Wie konnten Sie Ihre allgemeine Proposition entdecken?
Ich habe zuerst Varianten des ursprünglichen Integrals ausprobiert, indem ich Koeffizienten geändert habe. Mit inversen symbolischen Rechnern habe ich einige Muster gefunden . Dann habe ich versucht, einen schönen Parameter zu wählen, der das (vermutete) Ergebnis einfach aussehen lässt.

Für die Zwecke alternativer Methoden kann es von Interesse sein, darauf hinzuweisen, dass der Integrand

F ( X ) = 1 X 1 + X 1 X Protokoll ( 2 X 2 + 2 X + 1 2 X 2 2 X + 1 )
kann in Bezug auf hyperbolische trigonometrische Funktionen umgeschrieben werden. Verwenden
Tanh 1 ( z ) = 1 2 Protokoll ( 1 + z 1 z ) ,
und wir erhalten

F ( X ) = 1 X e Tanh 1 X Protokoll ( 1 + 2 X 1 + 2 X 2 1 2 X 1 + 2 X 2 ) = e Tanh 1 X ( 2 Tanh 1 ( 2 X 1 + 2 X 2 ) X ) .

Die rationale Funktion in der Klammer, die wir bezeichnen werden S ( X ) , ist ungefähr symmetrisch X = 0 .

Das gesuchte Integral ist

ICH = 1 1 F ( X ) D X = 1 1 e Tanh 1 X S ( X ) D X ,

was durch Hinzufügen des angegebenen nützlichen bestimmten Integrals zu beiden Seiten ergibt

ICH + 1 1 e Tanh 1 X S ( X ) D X = 2 1 1 S ( X ) D X 1 X 2 .

Verwenden Sie nun die Änderung der Variablen X = j wir haben

1 1 e Tanh 1 X S ( X ) D X = 1 1 e Tanh j S ( j ) D j = 1 1 e Tanh j S ( j ) D j = ICH ,
durch die Symmetrie von S ( X ) . Daher erhalten wir schließlich

ICH = 1 1 S ( X ) D X 1 X 2 = 2 1 1 1 X 1 X 2 Tanh 1 ( 2 X 1 + 2 X 2 ) D X .

Dieses Integral ist ungefähr symmetrisch X = 0 , also haben wir

ICH = 4 0 1 1 X 1 X 2 Tanh 1 ( 2 X 1 + 2 X 2 ) D X ,
die umgeschrieben werden können
ICH = 4 0 1 ( D D X sech 1 X ) Tanh 1 ( 2 X 1 + 2 X 2 ) D X .

Durch partielle Integration ergibt sich daraus

ICH = 8 0 1 sech 1 ( X ) ( 1 2 X 2 ) 1 + 4 X 4 D X .


Wir könnten auch die Änderung der Variablen vornehmen j = sech 1 X erhalten

ICH = 8 0 j ( cosch 2 ( j ) 2 ) Sünde j cosch 4 ( j ) + 4 D j = 8 0 j Sünde 3 j cosch 4 j + 4 D j 8 0 j Sünde j cosch 4 j + 4 D j .

Wie habe ich diese eine vor 0_0 großartige Lösung nicht gesehen

Diese Antwort bietet eine Möglichkeit zu finden ICH = 0 1 ln ( X 4 2 X 2 + 5 ) ln ( 5 X 4 2 X 2 + 1 ) 1 X 2   D X (die @RonGordon oben erhalten hat) mit Differenzieren unter dem Integralzeichen. A u -Ersetzung von u = 1 + X 2 1 X 2 ergibt dies.

ICH = 1 2 1 ln ( u 2 + 2 u + 2 u 2 2 u + 2 ) u 2 1   D u .
Integrieren Sie nun partiell mit A = ln ( u 2 + 2 u + 2 u 2 2 u + 2 ) Und D B = D u u 2 1 .
ICH = ln ( u 2 + 2 u + 2 u 2 2 u + 2 ) ln ( u + u 2 1 ) ] 1 + 2 1 u 2 2 u 4 + 4 ln ( u + u 2 1 )   D u
Der erste Term ist gleich 0 , also bleiben wir dabei.
ICH = 2 1 u 2 2 u 4 + 4 ln ( u + u 2 1 )   D u
Wir beginnen nun mit dem Schritt des Differenzierens unter dem Integral. Betrachten Sie das folgende Integral:
F ( A ) = A 1 X 2 A 2 X 4 + A 4 ln ( X + X 2 1 )   D X
Beachten Sie, dass trivialerweise F ( 0 ) = 0. Ein schneller u = X A ergibt dies.
F ( A ) = 1 A u 2 1 u 4 + 1 ln ( A u + ( A u ) 2 1 )   D u
Differenzierung bzgl A und unter Verwendung der Kettenregel erhalten wir dies.
F ' ( A ) = 1 × 1 A 2 × ( 1 A ) 2 1 ( 1 A ) 4 + 1 ln ( A ( 1 A ) + ( A ( 1 A ) ) 2 1 ) + 1 A X 2 1 X 4 + 1 × X ( A X ) 2 1   D X
Zum Glück fällt das erste Semester aus, also bleibt uns das übrig.
F ' ( A ) = 1 A X 2 1 X 4 + 1 × X ( A X ) 2 1   D X
A u -Ersetzung von u = ( A X ) 2 1 ergibt dies.
F ' ( A ) = 0 u 2 + 1 A 2 ( u 2 + 1 ) 2 + A 4   D u
Betrachten Sie das Integral mit u A 4 + 1 u
F ' ( A ) = 1 A 4 + 1 0 ( 1 A 2 ) u 2 + ( A 4 + 1 ) u 4 + 2 u 2 + ( A 2 + 1 )   D u
Wenn wir diese beiden Versionen des Integrals addieren und Zähler und Nenner des Integranden durch dividieren u 2 , erhalten wir folgendes.
F ' ( A ) = ( 1 A 2 ) + A 4 + 1 2 A 4 + 1 × 0 1 + A 4 + 1 u 2 ( u A 4 + 1 u ) 2 + 2 ( 1 + A 4 + 1 )   D u
Wir können endlich eine sehr schöne Substitution von durchführen w = u A 4 + 1 u dieses Integral zu lösen.
F ' ( A ) = ( 1 A 2 ) + A 4 + 1 2 A 4 + 1 × D w w 2 + 2 ( 1 + A 4 + 1 )   D w
Somit können wir das endlich sagen F ' ( A ) = ( 1 A 2 ) + A 4 + 1 2 A 4 + 1 × π 2 ( 1 + A 4 + 1 ) . Nach ein wenig beträchtlicher Algebra können wir das einfach erreichen, um dies zu erhalten.
F ' ( A ) = π 2 A 4 + 1 A 2 A 4 + 1
Integrierend können wir dies nun über den Wert von sagen F ( A ) .
F ( A ) = π 2 0 A X 4 + 1 X 2 X 4 + 1   D X
Einziger u -Ersetzung von u = X 4 + 1 X 2 ist hier erforderlich, um diese zu erhalten.
F ( A ) = π 2 2 A 4 + 1 A 2 1 D u 1 u 2
Dies ist natürlich gleich π arccos ( A 4 + 1 A 2 ) 2 2 .

Wir werden dieses Ergebnis nun in eine Funktion mit umwandeln arctan drin.

F ( A ) = π arccos ( A 4 + 1 A 2 ) 2 2 = π 2 arctan ( A 4 + 1 1 A 2 )

Unser gewünschter Wert für unser ursprüngliches Integral ist 2 F ( 2 ) .

0 1 ln ( X 4 2 X 2 + 5 ) ln ( 5 X 4 2 X 2 + 1 ) 1 X 2   D X = π arctan ( 5 1 2 ) = π arccot ϕ

Die endgültige Antwort auf das ursprüngliche Problem lautet also 4 π arccot ϕ .

Wir danken Ronaks Antwort auf Brilliant.org (veröffentlicht am 31. Dezember 2015).
@IshanSingh Ich habe diese Antwort nicht kopiert; Mir wurde das ursprüngliche Integral gegeben und anschließend der Anfangsbuchstabe u = 1 + X 2 1 X 2 von einem Freund, der mir diese Frage gestellt hat.
Sie können die Quadratwurzel mit entfernen 4 π Arccosec ϕ

Bemerkenswerterweise findet RIES ( http://mrob.com/pub/ries/index.html ) geschlossene Form aus Zahlenwert in Form einer Gleichung:

cos ( X π ) + 1 = 2 ϕ 6 .

Wenn wir oben vereinfachen, erhalten wir eine andere Form des Ergebnisses:

ICH = π arccos ( 17 8 5 ) .

Ich habe auch RIES verwendet, aber einige einfache Anpassungen vorgenommen, um eine einfachere Form der Antwort zu erhalten ICH = 8 π arcsin ( ϕ 1 ) .
@Somos Deine Antwort (ca 16.74 ) scheint doppelt so hoch zu sein wie die von Andrzej Odrzywolek und von Arcturus (ca 8.37 )
@Henry Hoppla! Danke. Die richtige Antwort ist ICH = 4 π arcsin ( ϕ 1 ) . Irgendwie habe ich es verdoppelt.

Dies ist nicht wirklich eine Antwort, aber grob zu lang für einen Kommentar. Ich wusste nicht, wie ich es über die endgültige Lösung hinaus vereinfachen sollte.

ICH = 1 1 1 X 1 + X 1 X ln ( 2 X 2 + 2 X + 1 2 X 2 2 X + 1 ) D X

Beginnen Sie mit der Substitution von X = cos 2 A

ICH = 1 1 1 cos 2 A 1 cos 2 A 1 + cos 2 A ln ( 2 cos 2 2 A 2 cos 2 A + 1 2 cos 2 2 A 2 cos 2 A + 1 ) D X

Durch die Tangenten- und cos-Doppelwinkeleigenschaften

ICH = 1 1 Sek 2 A | bräunen A | ln ( 2 cos 2 2 A + cos 4 A + 2 2 cos 2 A + cos 4 A + 2 ) D A

Waren gerade erst am Anfang. Jetzt ersetzen A = 1 2 gd ( B ) Wo gd ist die Gudermannsche Funktion.

ICH = 1 1 Sek ( gd ( B ) ) | bräunen ( gd ( B 2 ) ) | ln ( 2 cos 2 ( gd ( B ) ) + cos ( 2 gd ( B ) ) + 2 2 cos 2 ( gd ( B ) ) + cos ( 2 gd ( B ) ) + 2 ) D A

Hehe. Jetzt können wir etwas vereinfachen. Dies ist unter der Definition von Gudermannschen Eigenschaften.

ICH = 1 1 cosch   B | Sünde B 2 | ln ( 2 sech 2 B + ( sech 2 B + Tanh 2 B ) + 2 2 sech 2 B + ( sech 2 B + Tanh 2 B ) + 2 )

Verwenden Sie nun die Eigenschaften von Tanh Und sech noch weiter zu vereinfachen

ICH = 1 1 cosch   B | Sünde B 2 | ln ( ( 1 sech 2 B ) + 2 ( 1 + sech 2 B ) + 2 )

Unser Ziel ist es, eine arctanh funktionieren, aber das erfordert natürlich einige ernsthafte Anstrengungen. Faktorisieren Sie a 3 das zu generieren 1 benötigt, auch wenn es ein hässliches Factoring macht.

ICH = 1 1 cosch   B | Sünde B 2 | ln ( 3 ( 1 sech 2 B 3 ) 3 ( 1 + sech 2 B 3 ) )

Und jetzt schneiden Sie alle 3 aus. Verwenden Sie nach diesem Schnitt eine Eigenschaft von ln 's, um das Argument von zu erwidern ln . Und multipliziere 2 und 1/2

ICH = 1 1 2 cosch   B | Sünde B 2 | 1 2 ln ( ( 1 + sech 2 B 3 ) ( 1 sech 2 B 3 ) )

Und was weißt du! Du bist da! Verwenden Sie eine Eigenschaft von ln Und arctanh um eine viel SAUBERERe Form zu erzeugen (auch durch das Werfen der 2 nach vorne).

ICH = 2 1 1 cosch   B | Sünde B 2 | arctanh ( sech 2 B 3 )

Diese Funktion ist gerade, und wir können das wissen, weil alle Teile dessen, was oben ist, cosch B , | Sünde B | , usw. alles gleichmäßig. Wir können also Folgendes tun.

ICH = 4 0 1 cosch   B | Sünde B 2 | arctanh ( sech 2 B 3 )

Dies ist nur eine Idee und wie gesagt keine wirkliche Lösung. Ich habe keine Ahnung, wo ich darüber hinaus weitermachen soll, aber ich dachte, es könnte hilfreich sein, eine neue Idee zur Lösung zu finden.

Nach weiterer Inspektion habe ich meine Arbeit hier durcheinander gebracht. Ich werde diesen Beitrag jedoch hier lassen, da der Zweck des Beitrags immer noch besteht (Ideen zur Lösung)
Müssen Sie die Grenzwerte nicht ändern, nachdem Sie die erste Änderung der Variablen vorgenommen haben? X = cos 2 A ?
@ user23055 nicht wirklich. Es gibt viele Fehler und besteht nur aus Ersatz

Acht Jahre später.

Ausgehend von @Ron Gordons Substitution

8 0 ( u 2 1 ) ( u 4 6 u 2 + 1 ) u 8 + 4 u 6 + 70 u 4 + 4 u 2 + 1 Protokoll ( u ) D u
da die Wurzeln der Polynome in u 2 sind einfach, wir können Partialbruchzerlegung verwenden (ich werde die Formeln nicht tippen) und wir stehen vor vier Integralen
ICH = a β X 2 + γ Protokoll ( X ) D X
wobei alle Koeffizienten komplexe Zahlen sind. Dann
ICH = ich a ( Li 2 ( ich X β γ ) Li 2 ( ich X β γ ) + Protokoll ( X ) ( Protokoll ( 1 ich β X γ ) Protokoll ( 1 + ich β X γ ) ) ) 2 β γ
welche machen
J = 0 a β X 2 + γ Protokoll ( X ) D X = ich a ( Protokoll 2 ( ich β γ ) Protokoll 2 ( ich β γ ) ) 4 β γ = π a Protokoll ( β γ ) 4 β γ
Dies ergibt als Ergebnis
8 0 ( u 2 1 ) ( u 4 6 u 2 + 1 ) u 8 + 4 u 6 + 70 u 4 + 4 u 2 + 1 Protokoll ( u ) D u = π ( π Kinderbett 1 ( 1 4 22 + 17 5 ) )
Ich konnte nicht weiter vereinfachen.

Bearbeiten

Wenn Sie sich diese Frage von mir ansehen , hat @Jyrki Lahtonen die Vereinfachung vorgenommen, zu der ich nicht in der Lage war.

Ich dachte, ich würde einen Beitrag leisten und eine in sich geschlossene echte Analysemethode hinzufügen:

Ich werde die folgenden Darstellungen verwenden, die zum Beweis ziemlich einfach sind:

2 0 cos ( X ) cos ( T X ) X ( e X A B ich + e X A + B ich ) D X = ln ( ( T 2 A ) 2 + B 2 ) ln ( ( 1 A ) 2 + B 2 )
0 Sünde ( X ) X ( e X A B ich + e X A + B ich ) D X = arctan ( 1 A B ich ) + arctan ( 1 A + B ich )
0 cos ( T ) cos ( T X ) X 2 1 D X = π 2 Sünde ( T ) , für T 0

Jetzt können wir mit der Auswertung beginnen ICH :

ICH = 1 1 1 X 1 + X 1 X ln ( 2 X 2 + 2 X + 1 2 X 2 2 X + 1 ) D X

Erzwingen Sie die Substitution 1 + X 1 X = u :

ICH = 0 4 u 2 u 4 1 ln ( 5 u 4 2 u 2 + 1 u 4 2 u 2 + 5 ) D u = u 1 u 0 4 u 4 1 ln ( 5 u 4 2 u 2 + 1 u 4 2 u 2 + 5 ) D u

Aus der Addition der beiden können wir ableiten:

ICH = 0 2 u 2 1 ln ( 5 u 4 2 u 2 + 1 u 4 2 u 2 + 5 ) D u

Seit

ln ( 5 u 4 2 u 2 + 1 ) ln ( 4 ) = ln ( ( u 2 1 5 ) 2 + 4 25 ) ln ( 4 5 )
ln ( u 4 2 u 2 + 5 ) ln ( 4 ) = ln ( ( u 2 1 ) 2 + 4 ) ln ( 4 )

Wir können schreiben

A = 0 2 u 2 1 ( ln ( ( u 2 1 5 ) 2 + 4 25 ) ln ( 4 5 ) ) D u B = 0 2 u 2 1 ( ln ( ( u 2 1 ) 2 + 4 ) ln ( 4 ) ) D u
so dass ICH = A B

Nun leiten wir aus den Ausgangsdarstellungen ab:

A = 0 4 u 2 1 0 cos ( T ) cos ( u T ) T ( e T ( 1 2 ich ) / 5 + e T ( 1 + 2 ich ) / 5 ) D T D u = 2 π 0 Sünde ( T ) T ( e T ( 1 2 ich ) / 5 + e T ( 1 + 2 ich ) / 5 ) D T = 2 π arctan ( 5 1 2 ich ) + 2 π arctan ( 5 1 + 2 ich ) = ( 4 π arctan ( 5 1 2 ich ) )

Ähnlich findet man

B = ( 4 π arctan ( 1 1 2 ich ) )

Durch die Verwendung der Identität:

arctan ( X ) + arctan ( j ) = arctan ( X + j 1 X j )

Man folgert

ICH = 4 π arccot ( ϕ )